Block 2 Flashcards

1
Q

A 67 year old man presents to the emergency department with shaking chills and a temperature of 101F. Laboratory examination reveals a hematocrit of 23%, and urine tests are positive for blood. The patient states that he is taking only one medication for his irregular heartbeat. Which of the following drugs most likely caused the appearance for these symptoms in this patient?

A. Verapamil
B. Digoxin 
C. Propanolol
D. Quinidine 
E. Hydralazine
A

D. Quinidine (with hemolytic anemia)

How well did you know this?
1
Not at all
2
3
4
5
Perfectly
2
Q

An elderly man presents with complaints of ringing in his ears, blurred vision, and upset stomach. He is taking multiple medications. His wife states that he has few episodes of confused, delirious behavior over the past few weeks. Which of the following agents might be responsible for this man’s symptons?

A. Allopurinol 
B. Hydralazine
C. Spironolactone 
D. Quinidine 
E. Niacin
A

D. Quinidine

CNS symptoms

How well did you know this?
1
Not at all
2
3
4
5
Perfectly
3
Q
Figure below illustrates a current concept of the control of gastric acid secretion. Which of the following drugs acts at the site labeled "ATPase"? 
A. aluminum hydroxide 
B. misoprostol 
C. omeprazole 
D. ranitidine 
E. sucralfate
A

C. Omeprazole

Proton pump inhibitors

How well did you know this?
1
Not at all
2
3
4
5
Perfectly
4
Q

A 35 year old woman appears to have familial combined hyperlipidemia. Her serum concentrations of total cholesterol, LDL cholesterol, & triglyceride are elevated. Her serum concentration of HDL cholesterol is somewhat reduced. Which of the following drugs is most likely to cause an increase in this patient’s TG and VLDL cholesterol when used as monotherapy?

A. Atorvastatin 
B. Gemfibrozil 
C. Cholestyramine 
D. Niacin 
E. All of the above
A

C. Cholestyramine

Bile binding

How well did you know this?
1
Not at all
2
3
4
5
Perfectly
5
Q

A 52 year old man with peptic ulcer disease has been on drug therapy for 3 months and has noticed changes in his bowel habits, increasing headaches, dizziness, skin rashes, loss of libido and gynecomastia. Which of the following drugs is most likely responsible for these side effects?

A. Cimetidine 
B. Metronidazole 
C. Sucralfate 
D. Omeprazole 
E. Ranitidine
A

A. Cimetidine

How well did you know this?
1
Not at all
2
3
4
5
Perfectly
6
Q

A 58 year old business executive is brought to the emergency room 2 hours after the onset of severe chest pain during a vigorous tennis game. She has a history of poorly controlled mild hypertension and elevated blood cholesterol but does not smoke. ECG changes cnfirm the diagnosis of myocardial infaraction. The decision is made to attempt to open her occluded artery. Conversion of plasminogen to plasmin is brought about by

a. Heparin
b. Warfarin
c. Reteplase
d. Aminocaproic acid
e. Lepirudin

A

c. Reteplase

Only tPa

How well did you know this?
1
Not at all
2
3
4
5
Perfectly
7
Q

A patient with multiple medical problems is taking several drugs, including theophylline, warfarin, quinidine, and phenytoin. Despite the likelihood of interactions, dosages of each are adjusted carefully so their serum concentrations and effects are acceptable. However, the patient suffers some GI distress and starts taking a drug provided by one of his “well intentioned” friends. He presents with excessive or toxic effects from all his other medications and blood tests reveal that their serum concentrations are high. Which was the drug the patient most likely self-prescribed and took?

A. Nizatidine 
B. Sucralfate 
C. Ranitidine 
D. Famotidine 
E. Cimetidine
A

E. Cimetidine

How well did you know this?
1
Not at all
2
3
4
5
Perfectly
8
Q

***A 62 year old white man complains of left thigh and leg pain and swelling that are exacerbated by walking. One week earlier, the patient underwent cardiac catheterization. The patient is currently vacationing and has spent the past 28 hours in a car. Which of the following drugs, which might be prescribed in this instance, works by inhibiting the enzyme epoxide reductase?

A. Tissue-type plasminogen activator (tPa) 
B. Dipyridamole
C. Heparin 
D. Streptokinase 
E. Acetylsalicylic acid 
F. Warfarin
A

F. Warfarin

Warfarin affects INR in blood

How well did you know this?
1
Not at all
2
3
4
5
Perfectly
9
Q

***A 67 year old woman with Parkinson disease responds well to treatment with a combination of levodopa and carbidopa. The woman develops gastroesophageal reflux disease (GERD) and is prescribed an appropriate drug for its management. She is instructed to continue talking levodopa and carbidopa.. After she starts taking the drug for GERD, she complains that her symptoms of Parkinson disease have gotten worse. The drug most likely prescribed to treat GERD in this patient was

A. famotidine 
B. bethanechol
C. cisapride
D. metoclopramide 
E. atropine
A

D. metoclopramide

How well did you know this?
1
Not at all
2
3
4
5
Perfectly
10
Q

If a patient undergoes a percutaneous coronary procedure and placement of a stent in a coronary blood vessel, she may be given EPTIFIBATIDE. The mechanism of EPTIFIBATIDE’s anticlotting action is

A. Reversible inhibition of glycoprotein IIb/IIIa receptors
B. Blockade of posttranslational modification of clotting factors
C. Inhibition of thromboxane production
D. Activation of antithrombin III
E. Irreversible inhibition of platelet ADP receptors

A

A. Reversible inhibition of glycoprotein IIb/IIIa receptors

How well did you know this?
1
Not at all
2
3
4
5
Perfectly
11
Q

A 58 year old woman is being treated for chronic suppression of a ventricular arrhythmia. After two months of therapy, she complains about feeling tired all the time. Examination reveals a resting heart rate of ten beats per minute lower than her previous rate. Her skin is cool and clammy. Laboratory test results indicate low thyroxin and elevated thyroid-stimulating hormone levels. Which of the following antiarrhythmic drugs is the likely cause of these signs and symptoms?

A. Amiodarone 
B. Quinidine
C. Propanolol 
D. Verapamil 
E. Procainamide
A

A. Amiodarone

How well did you know this?
1
Not at all
2
3
4
5
Perfectly
12
Q

A 64 year old man presents to his physician with aching, burning pain after meals. He has been self medicating with antacids for several months but has found this to be increasingly ineffective. His physician decides to take him off the antacids and instead places him on a combination of Ranitidine and sucralfate. Why is this combination a bad idea?

A. Ranitidine increases the toxicity of sulcralfate
B. Ranitidine inhibits the action of sucralfate
C. Sucralfate and ranitidine co precipitate
D. Sucralfate increases the toxicity of ranitidine
E. Sulcralfate inhibits the actions of Ranitidine

A

B. Ranitidine inhibits the action of sucralfate

How well did you know this?
1
Not at all
2
3
4
5
Perfectly
13
Q

Which of the following drugs reduces LDL cholesterol by inhibiting an intestinal transport protein?

A. Atorvastatin 
B. Cholestyramine 
C. Gemfibrozil 
D. Ezetimibe 
E. Niacin
A

D. Ezetimibe

How well did you know this?
1
Not at all
2
3
4
5
Perfectly
14
Q
A 25 year old woman comes to her family physician for a routine check-up. Her physical examination shows a mildly overweight woman but is otherwise unremarkable. A fasting lipid panel, however, shows an LDL cholesterol level of 310 mg/dL, HDL cholesterol level of 42 mg/dL, triglyceride level of 150 mg/dL, and total cholesterol level of 382 mg/dL. Because a diagnosis of familial hpercholesterolemia is suspected, the doctor initiates treatment of her condition. Soon after starting treatment, however, she presents with myalgias. Laboratory values show elevated levels of aspartate aminotransferase, alanine aminotransferase, and creatinine kinase. Which of the following interventions is most likely responsible for the patient's myalgias?
A. B complex vitamins 
B. hormone replacement therapy 
C. Liver transplant 
D. Low HDL 
E. Statin Medications
A

E. Statin Medications

How well did you know this?
1
Not at all
2
3
4
5
Perfectly
15
Q
A 35 year old woman appears to have familial combined hyperlipidemia. Her serum concentrations of total cholesterol, LDL cholesterol, and triglyceride are elevated. Her serum concentration of HDL cholesterol is somewhat reduced. If this patient is pregnant, which of the following drugs should be avoided because of a risk of harming the fetus?
A. Cholestyramine 
B. Cholestipol 
C. Pravastatin 
D. Niacin 
E. Vitamin B3
A

C. Pravastatin

NO STATINS IN PREGNANCY

How well did you know this?
1
Not at all
2
3
4
5
Perfectly
16
Q

A 20 year old varsity hockey player is referred to you by his coach. The young athlete has excessive bruising after a very physical match 2 days before. His knee had been bothering him, so he took two 325-mg aspirin tablets several hours before the contest. He got checked hard into the boards. 10 times during the game, but denies any excessive or unusual trauma. As you ponder the etiology you order several blood tests. Which test or finding do you most likely expect to be abnormal as a result of the prior aspirin use?

A. Platelet count 
B. Bleeding time 
C. INR (International normalized ratio) 
D. Activated partial thromboplastin time (APTT) 
E. Prothrombin time
A

B. Bleeding time (prolonged)

How well did you know this?
1
Not at all
2
3
4
5
Perfectly
17
Q

***A patient with a supraventricular tachycardia has an atrial rate of of 280/min with a ventricular rate of 140/min via a 2:1 AV nodal transmission. After treatment with a drug, the atrial rate slowed to 180/min, but the ventricular rate increased to 180/min. Which of the following drugs was most likely to have been given to this patient?

A. Adenosine 
B. Quinidine 
C. Digoxin 
D. Esmolol 
E. Verapamil
A

B. Quinidine (increase in AV conduction)

How well did you know this?
1
Not at all
2
3
4
5
Perfectly
18
Q

A patient develops severe thrombocytopenia in response to treatment with unfractionated heparin and still requires parenteral anti coagulation. The patient is most likely to be treated with

A. Abciximab 
B. Lepuridin 
C. Streptokinase 
D. Urokinase 
E. Plasminogen
A

B. Lepuridin

How well did you know this?
1
Not at all
2
3
4
5
Perfectly
19
Q

***A 22 year old woman who experienced pain and swilling in her right leg presented at the emergency room. An ultrasound study showed thrombosis in the popliteal vein. The patient, who was in her second trimester of pregnancy, was treated for seven days with intravenous unfractionated heparin. The pain resolved during the course of therapy, and the patient was discharged on day eight. Which one of the following drugs would be most appropriate outpatient follow-up therapy for this patient, who lives 100 miles from the nearest hospital?

A. Streptokinase 
B. Warfarin 
C. LMWH 
D. Unfractionated heparin 
E. Alteplase
A

C. LMWH

Low molecular weight heparin

How well did you know this?
1
Not at all
2
3
4
5
Perfectly
20
Q

Cardiac output is a function of stroke volume and heart rate. Stroke volume increases when contractility increases, preload increases, or afterload decreases. There are a number of factors that affect each of these components and ultimately cardiac output. Which of the following variations woul increase cardiac output?

A. Atenolol treatment 
B. Verapamil treatment 
C. Clonidine 
D. Adenosine 
E. Cardiac Glycoside administration
A

E. Cardiac Glycoside administration

Digitalis group

How well did you know this?
1
Not at all
2
3
4
5
Perfectly
21
Q

A 67 year old woman is brought to the emergency department with complaints of fever, malaise and the recent appearance of a malar “butterfly” rash on the face. The patient states that she is taking one medication for arrhythmias. If she is positive for antihistone and is mildly anemic, which of the following medications is she most likely taking?

A. Mexiletine 
B. Disopyramide 
C. Flecainide 
D. Procainamide 
E. Digoxin
A

D. Procainamide

How well did you know this?
1
Not at all
2
3
4
5
Perfectly
22
Q

A 29 year old man has had episodes of watery diarrhea and abdominal pain for the past year. He has been defecating as many as 10 times a day during the past week, nd he has lost about 8 kg (18 lb) during the past 3 months. His temperature is 38.3 degrees C (101 F). Laboratory studies and sigmoidoscopy findings are consistent with the diagnosis of severe acute ulcerative colitis. Which of the following drugs would be most appropriate to administer as an enema for short-term treatment of acute exacerbations of ulcerative colitis in this patient?

A. Infliximad 
B. Cyclosporine 
C. Amoxicillin 
D. Hydrocortisone 
E. Sulfasalazine
A

D. Hydrocortisone

How well did you know this?
1
Not at all
2
3
4
5
Perfectly
23
Q

***Which one of the following drugs causes a decrease in liver triacylglycerol synthesis by limiting available free fatty acids needed as building blocks for this pathway?

A. Probucol 
B. Niacin 
C. Cholestyramine 
D. Lovastatin 
E. Fenofibrate
A

B. Niacin or Nicotinic acid

How well did you know this?
1
Not at all
2
3
4
5
Perfectly
24
Q

A 67 year old woman with a history of venous thromboembolism is placed on warfarin (Coumadin) prophylactically. The blood concentration of Coumadin becomes too high and bleeding occurs. This bleeding can best be treated by the administration of which of the following?

A. Fibrinogen 
B. Thrombin 
C. Platelets 
D. Protein C 
E. Vitamin K
A

E. Vitamin K

How well did you know this?
1
Not at all
2
3
4
5
Perfectly
25
Q

A 41 year old woman who is diabetic presents to her physician complaining of GI distress and bloating particularly after meals. Which of the following drugs is useful to her?

A. Sucralfate 
B. Metoclopramide 
C. Famotidine 
D. Omeprazole 
E. Ranitidine
A

B. Metoclopramide

How well did you know this?
1
Not at all
2
3
4
5
Perfectly
26
Q

If quinidine and digoxin are administered concurrently, which of the following effects does quinidine have on digoxin?

A. The absorption of digoxin form the GI tract is decreased
B. The effect of digoxin on the AV node is antagonized
C. The concentration of digoxin in the plasma is increased
D. The metabolism of digoxin is prevented
E. The ability of digoxin to inhibit the Na+K+ stimulated ATPase is reduced

A

C. The concentration of digoxin in the plasma is increased

How well did you know this?
1
Not at all
2
3
4
5
Perfectly
27
Q

A 44 year old obese man has extremely high plasma triglyceride levels, but cholesterol levels are within normal limits. Following treatment with a drug specifically indicated for hypertriglyceridemia, triglyceride levels decrease to almost normal. Which of the following agents is most likely to have caused this desired change?

A. Atorvastatin 
B. Cholestyramine 
C. Ezitemibe 
D. Colestipol 
E. Gemfibrozil
A

E. Gemfibrozil

Gemfibrozil - “fibrate” - clofibrate, ferofibrate +LPL Decrease TG

How well did you know this?
1
Not at all
2
3
4
5
Perfectly
28
Q

A patient in the coronary care unit develops episodes if paroxysmal AV nodal reentrant tachycardia (PSVT). Which of the following would generally be considered a first-line drug for promptly stopping the arrhythmia?

A. Adenosine 
B. Edrophonium 
C. Digoxin 
D. Phenylephrine 
E. Propranolol
A

A. Adenosine

How well did you know this?
1
Not at all
2
3
4
5
Perfectly
29
Q

A couple celebrating their fortieth wedding anniversary is given a trip to Peru to visit Machu Picchu. Due to past experiences while traveling, they ask their doctor to prescribe an agent for diarrhea. Which of the following would be effective?

A. Lorazepam
B. Omeprazole 
C. Famotidine 
D. Loperamide 
E. All of the above
A

D. Loperamide

How well did you know this?
1
Not at all
2
3
4
5
Perfectly
30
Q

A patient who has been a high-dose alcohol abuser for many years presents with hepatic portal-stsemic encephalopathy. Which of the following drugs, give in relatively high doses, would be most suitable for the relief of signs and symptoms of this condition, and the likely underlying biochemical anomalies?

A. Lactulose 
B. Esomeprazole 
C. Diphenoxylate 
D. Ondansetron 
E. Loperamide
A

A. Lactulose

Binds to ammonia ion and excretes it

How well did you know this?
1
Not at all
2
3
4
5
Perfectly
31
Q

***Following a myocardial infaraction, a patient in the emergency room of a hospital develops ventricular tachycardia. The best way to manage this situation is with the administration of

A. Dilitazem 
B. Flecainide 
C. Amiodorone 
D. Lidocaine 
E. Adenosine
A

D. Lidocaine

How well did you know this?
1
Not at all
2
3
4
5
Perfectly
32
Q

A patient is admitted for acute coronary syndrome with signs of impending myocardial infarction. She undergoes angioplasty with double coronary stenting to maintain the patency of her coronary vessels. Which of the following drugs will probably be administered to prevent clotting in the region of the wire mesh stents?

A. Dilitazem 
B. Flecainide 
C. Amiodorone 
D. Lidocaine 
E. Adenosine
A

??

Clopidogrel

The primary process in the formation of this arterial clot is platelet aggregation, so an antiplatelet drug such as clopidogrel is suitable

or Lidocaine (if it is the option)

How well did you know this?
1
Not at all
2
3
4
5
Perfectly
33
Q

A 21 year old man is brought by his roommate to the emergency department because of abrupt onset of shortness of breath, mild chest pain and sensation of rapid heart beating. ECG documents. Supraventricular arrhythmias wiht pulse of 200 per minute. Under ECG monitoring gentle massage over right carotid sinus is attempted but attack doesn’t cease. Which of the following is most appropriate next step of managaement?

A. IV Lignocaine 
B. Further carotid massage 
C. IV Procainamide 
D. IV Verapamil 
E. IV Amlodipine
A

D. IV Verapamil (or diltiazem)

ABCD - Calcium Channel Blocker

How well did you know this?
1
Not at all
2
3
4
5
Perfectly
34
Q

A 68 year old woman with a history of coronary insufficiency suffers a heart attack and begins taking anticoagulant. One year later, her physician prescribes colestyramine to treat her hypercholesterolemia. A few months after the patient begins colestyramine treatment, she develops gastrointestinal bleeding and is found to have hypothrombinemia. Which of the following mechanisms accounts for how colestyramine causes hypoprothrombinemia?

A. Distribution 
B. Absorption 
C. competes with plasma proteins 
D. Metabolism 
E. Pharmacodynamics
A

B. Absorption

How well did you know this?
1
Not at all
2
3
4
5
Perfectly
35
Q

***A 63 year old woman develops tachyarrhythmia after recovering from an MI. Her physician prescribes a new medication. Five montgs later she presents to the clinic complaining of difficulty breathing and dry coughing. Neither of which she has experienced previously. Physical examination reveals decreased breath sounds. After ruling out cardiac causes for her symptoms, her physician conducts a series of tests. X ray of the chest shows decreased lung volume. Lung function tests show decreased vital capacity. Which agent most likely caused her current condition?

A. Amiodarone 
B. Procainamide 
C. Quinidine 
D. Verapamil 
E. Proponolol
A

A. Amiodarone

Restrictive lung disease

Amiodarone - thyroid dysfunction, causes pulmonary fibrosis and smurf skn.

How well did you know this?
1
Not at all
2
3
4
5
Perfectly
36
Q

A patient with peptic ulcer disease is taken off their medication because of undesirable side effects. As a result, the patient has rebound gastric acid hypersecretion. Which of the following drugs best accounts for the observed result?

a. An H 1 -receptor antagonist
b. A proton pump inhibitor
c. A cholinergic receptor antagonist
d. An antacid
e. A CCKB receptor antagonist

A

D. An antacid

acid rebound

How well did you know this?
1
Not at all
2
3
4
5
Perfectly
37
Q
  • **We have a 28-year-old female patient with Stage II essential hypertension, tachycardia, and occasional palpitations (ventricular ectopic beats). Normally we might consider prescribing a beta blocker to control the blood pressure and cardiac responses, but our patient also has asthma, and she is trying to get pregnant. Which of the following drugs would be the best alternative to the beta blocker in terms of likely efficacy on pressure and heart rate, and in terms of relative safety?
    a. Diltiazem
    b. Enalapril
    c. Furosemide
    d. Phentolamine
    e. Prazosin
A

A. Diltiazem(calcium channel blocker)

Asthma, no non selective B Blocker

How well did you know this?
1
Not at all
2
3
4
5
Perfectly
38
Q

***An elderly male patient who has just been referred to your practice has been taking a drug for symptomatic relief of benign prostatic hypertrophy.
In addition to its effects on smooth muscles of the prostate and urethra, this drug can lower blood pressure in such a way that it reflexly triggers tachycardia, positive inotropy, and increased AV nodal conduction. The drug neither dilates nor constricts the bronchi. It causes the pupils of the eyes to constrict and interferes with mydriasis in dim light. Initial oral dosages of this drug have been associated with a high incidence of syncope. Which prototype is most similar to this unnamed drug in terms of the pharmacologic profile?
a. Captopril
b. Hydrochlorothiazide (prototype thiazide diuretic)
c. Labetalol
d. Nifedipine
e. Prazosin
f. Propranolol
g. Verapamil

A

E. Prazosin

Alpha 1 beta blockers - BPH (Benign prostatic hypertrophy

How well did you know this?
1
Not at all
2
3
4
5
Perfectly
39
Q

***We have just diagnosed Stage 1 essential hypertension in a 30-year old
man who has a history of asthma. He regularly uses an inhaled corticosteroid,
which seems to work well, but does need to use an albuterol inhaler about once every 3 weeks for suppression of asthma attacks. Which antihypertensive drug or drug class poses the greatest risk of exacerbating the patient’s asthma and counteracting the desired pulmonary effects of the albuterol, even though it might control his blood pressure well?
a. Diltiazem
b. Hydrochlorothiazide
c. Labetalol
d. Ramipril
e. Verapamil

A

c. Labetalol (increases asthmatic affects, non selective Beta Blocker )

Asthma, stage 1 HTN,

How well did you know this?
1
Not at all
2
3
4
5
Perfectly
40
Q

A 56 year old woman with a long history of untreated hypertension brought to emergency department because of severe headache, confusion. There is disorientation to person but not to time and place. BP is 230/180, Pulse 86/min, respiratory rate is 18/min. Fundoscopic examination reveals optic disc edema and dipstick test shows protein in urine. Which of the following is most appropriate pharmacotherapy?

A. ACEI
B. Proponolol 
C. Nifedipine 
D. Sodium nitroprusside 
E. Furosemide
A

D. Sodium Nitroprusside

Protein in urine, malignant hypertension

How well did you know this?
1
Not at all
2
3
4
5
Perfectly
41
Q

A diabetic male presents with hypertension and 24-h urine showing 200 mg of albumin. In a diabetic patient with microalbuminuria, the approproate drug for treatment of hypertension to prevent progression of renal failure is

A. Beta blocker 
B. Thiazide diuretic 
C. Enalapril 
D. Short-acting dihydropyridine calcium channel blocker for precise control
(nifedipine)
E. Sodium Nitroprusside
A

C. Enalapril (angiotensin converting enzyme inhibitors

Diabetes with HTN

How well did you know this?
1
Not at all
2
3
4
5
Perfectly
42
Q

A 52 year old man with a 30 pack year history of cigarette smoking presents to a physician. He was told that he has high cholesterol about 2 years ago, and he has history of mild HTN for which he has never been treated. He had MI 6 months ago. His post MI has been uncomplicated. His medications include one aspirin tablet every other day. Physical examination is normal except 4th heart sound. Which of the following is most appropriate next step in management to prevent morbidity and mortality?

A. Add beta blocker 
B. Add diuretics 
C. Add calcium channel blocker 
D. Increase aspirin dose three times daily 
E. Prescribe nitroglycerin for angina
A

A. Add beta blocker

*Morbidity and mortality

How well did you know this?
1
Not at all
2
3
4
5
Perfectly
43
Q
  • **We have a patient with newly diagnosed essential hypertension, and start them on a commonly used antihypertensive drug at a dose that is considered to be therapeutic for the vast majority of patients. Soon after starting therapy the patient experiences crushing chest discomfort. EKG changes show myocardial ischemia. Studies in the cardiac cath lab show episodes of coronary vasospasm, and it is likely the antihypertensive drug provoked the vasoconstriction. Which of the following antihypertensive drugs or drug class most likely caused the ischemia and the angina?
    a. Atenolol
    b. Diltiazem
    c. Hydrochlorothiazide
    d. Losartan
    e. Metolazone
A

A. Atenolol or Proponolol

(No beta blockers in vasospastic angina)
Anti HTN drug cause ischemia and angina - non selective B blocker (exacerbate vasoconstriction)

How well did you know this?
1
Not at all
2
3
4
5
Perfectly
44
Q

***A patient is hospitalized and waiting for coronary angiography. His history includes angina pectoris that is brought on by “modest” exercise, and is accompanied by transient electrocardiographic changes consistent with myocardial ischemia. There is no evidence of coronary vasospasm. In
the hospital he is receiving nitroglycerin and morphine (slow intravenous infusions), plus oxygen via nasal cannula.
He suddenly develops episodes of chest discomfort. Heart rate during these episodes rises to 170-190 beats/min; blood pressure reaches 180-200/110-120 mm Hg, and prominent findings on the EKG are runs of ventricular ectopic beats that terminate spontaneously, plus ST-segment elevation.
Although there are several things that need to be done for immediate care, administration of which one of the following is most likely to remedy (at least temporarily) the majority of these signs and symptoms and pose the lowest risk of doing further harm?
a. Aspirin
b. Captopril
c. Furosemide
d. Labetalol
e. Lidocaine
f. Nitroglycerin (increased dose as a bolus)
g. Prazosin

A

C. Labetalol (decrease HR and decrease BP because it has beta and alpha blocking properties)

Given its combination of both α/β blocking effect, it is the best approach for managing the hypertension, the tachycardia, oxygen supply-demand imbalance that leads to both CP and the ischemic ST-changes, and the ventricular ectopy - which is probably a reflection of excessive catecholamine stimulation of β1
receptors

How well did you know this?
1
Not at all
2
3
4
5
Perfectly
45
Q

A hypertensive patient has been on long-term therapy with lisinopril for hypertension. The drug isn’t controlling pressure as well as wanted, so the physician decides to add Spirinolactone as the (only) second drug. Which of the following is the most likely outcome of adding this diuretic to the ACE inhibitor regimen?

A. Blood pressure would rise abruptly
B. Better BP control, but with a risk of hyperkalemia
C. Cardiac depression, because both drugs directly depress the heart
D. Cough that may be severe, even though there was no cough with lisinopril alone
E. Hypernatremia, because ACE inhibitors counteract triamterene’s natriuretic effect

A

B. Better BP control, but with a risk of hyperkalemia

How well did you know this?
1
Not at all
2
3
4
5
Perfectly
46
Q

A 55 year old male patient comes to his primary care physician for his routine health care examination. He is completely asymptomatic and has nothing significant in his past medical history. He has 10 pack years of cigarette smoking but he does not drink. His PR:80/min; BP: 150/78 mm Hg; Temperature: 37.1C(98.8F). Labs show hematocrit of 43%, blood glucose of 90 mg/dl, serum creatinine of 0.7 mg/dL, serum Na of 140 mEq/L and serum K of 4 mEq/L. Urinalysis and EKG are unremarkable. His blood pressure readings on subsequent visits are 155/80 mmHg, 160/78 mmHg and 150/70 mmHg. Life-style modifications fail to contrl his blood pressure. Which of the following is the best initial pharmacological therapy for the control of his blood pressure?

A. Thiazide diuretics 
B. Hydralazine 
C. Sodium Nitroprusside 
D. Angiotensin Receptor Blockers 
E. Sodium channel blockers
A

A. Thiazide Diuretics

How well did you know this?
1
Not at all
2
3
4
5
Perfectly
47
Q

A 55 year old African American female presents to the ER with lethargy and blood pressure of 250/150. Her family members indicate that she was complaining of severe headache and visual disturbance earlier in the day. They report a history severe asthma but no known kidney disease. On physical exam, papilledena and retinal hemorrhages are present. The best approach is

A. Intravenous labetalol therapy
B. Continuous-infusion nitroprusside
C. Clonidine by mouth to lower blood pressure slowly but surely
D. Diltiazem sublingually to lower blood pressure rapidly and remove the and remove the patient from danger
E. Further history about recent home antihypertensives before deciding current
therapy

A

b. Continuous-infusion nitroprusside

Malignant HTN - nitroprusside or labetolol (not with asthma)

How well did you know this?
1
Not at all
2
3
4
5
Perfectly
48
Q

A patient presents for treatment of his severe essential HTN. He is being treated with numerous medications for high blood pressure, and hydralazine was recently added to his medication regimen. He explains that he has been experiencing flushing and headaches since his last visit, when hydralazine therapy was started. Which of the following is an adverse effect of hydralazine?

A. Anginal attack 
B. Bradycardia 
C. First-dose orthostatic hypotension 
E. Nephrotoxicity 
F. Pulmonary embolism
A

A. Anginal attack, orthostatic HTN if given

Adverse effect of hydralazine

How well did you know this?
1
Not at all
2
3
4
5
Perfectly
49
Q

***A 48 year old hypertensive patient has been sucessfully treated with a thiazide diuretic for the last five years. Over the last three months, his diastolic pressure has steadily increased, and he has been started on an additional hypertensive medication. He complains of several instances of being unable to achieve an erection and that he is no longer able to complete three sets of tennis. The second anyhypertensive medication is most likelt which one of the following?

A. Captopril 
B. Losartan 
C. Minoxidil
D. Metoprolol
E. Nifedipine
A

D. Metoprolol

Beta blockers - decrease in sexual dysfunction

How well did you know this?
1
Not at all
2
3
4
5
Perfectly
50
Q

A patient with a history of hypertension, heart failure, and peripheral vascular disease has been on oral therapy with drugs suitable for each for about 3 months. He runs out of the medication and plans to have the prescriptions refilled in a week or so.
Within a day or two after stopping his medications he experiences an episode of severe tachycardia accompanied by tachyarrhythmias, and an abrupt rise of blood pressure to 240/140 mm Hg—well above pretreatment levels. He complains of chest pain, anxiety, and a pounding headache. Soon thereafter he suffers a hemorrhagic stroke.
Which of the following drugs or drug groups, the man suddenly stops taking, most likely causes these responses?
a. ACE inhibitors
b. Clonidine
c. Digoxin
d. Furosemide
e. Nifedipine (a long-acting formulation)
f. Warfarin

A

b. Clonidine

Rebound HTN - Hemorrhagic stroke if suddenly stopping medication

How well did you know this?
1
Not at all
2
3
4
5
Perfectly
51
Q

A 32 year old woman with pheochromocytoma is being treated with phenoxybenzamine. After surgical excision of the tumor, the patient has an episode of hypotension requiring 30 seconds of cardiopulmonar resusciation and subsequen treatment in the intensive care unit. The attending physician asks his intern what physiologic responses during resuscitation. What would have been observed following administration of epinephrine?

A. Decrease in blood pressure 
B. Decrease in heart rare
C. Increase in blood pressure 
D. Increase in respiratory rate 
E. No change in vital signs
A

A. Decrease in Blood Pressure (Epinephrine reversal phenomenon)

Pheochromocytoma - alpha blocker

1st administer Epinephrine -> increase BP
Alpha Blocker X -> Decrease BP
2nd administer Epinephrine -> Decease in BP (Beta 2 activation)

How well did you know this?
1
Not at all
2
3
4
5
Perfectly
52
Q

What type of drug may cause a precipitous fall in blood pressure and fainting on the initial administration.

A

Prazosin

How well did you know this?
1
Not at all
2
3
4
5
Perfectly
53
Q

Which drug produces headaches, palpitations, flushing and peripheral edema as side effects?

A

Dihydropyridine calcium channel blockers

How well did you know this?
1
Not at all
2
3
4
5
Perfectly
54
Q

Which antihypertensive drugs can precipitate hypertensive crisis following abrupt cessation of therapy?

A

Clonidine

How well did you know this?
1
Not at all
2
3
4
5
Perfectly
55
Q

The 45-year-old man has recently been diagnosed with hypertension and started on monotherapy designed to reduce peripheral resistance and prevent sodium and water retention. He has developed a persistent cough. Which drug is most likely responsible for this side effect?

A

Losartan

How well did you know this?
1
Not at all
2
3
4
5
Perfectly
56
Q

Which of the following beta blockers reduce after load?

A) Metoprolol
B) Propranolol
C) Carvedilol
D) Atenolol

A

C. Carvedilol

How well did you know this?
1
Not at all
2
3
4
5
Perfectly
57
Q

What is the most likely mechanism by which losartan lowers blood pressure?

A

Prevents the cardiovascular effects of angiotensin II mediated by AT-1 receptors

How well did you know this?
1
Not at all
2
3
4
5
Perfectly
58
Q

Which drug is the first line treatment for an African-American patient with hypertension and diabetic nephropathy?

A

Enalapril

How well did you know this?
1
Not at all
2
3
4
5
Perfectly
59
Q

We treat a patient with a drug that affects the clotting-thrombolytic systems for a time sufficient to let the drug’s effects and blood levels stabi- lize at a therapeutic level. We then isolate platelets from a blood sample and test their in vitro aggregatory responses to ADP, collagen, PAF, and throm- boxane A2. Aggregatory responses to ADP are inhibited; responses to the other platelet proaggregatory agonists are unaffected. Which drug did we most likely administer to this patient?

a. Aspirin
b. Bivalirudin
c. Clopidogrel
d. Heparin
e. Warfarin

A

c. Clopidogrel

How well did you know this?
1
Not at all
2
3
4
5
Perfectly
60
Q

You are reviewing the medication history of a 59-year-old man. He has been taking ramipril, pravastatin, and metformin for the last 5 years; and escitalopram for the last 12 months to help manage his depression. At his last clinic visit, a year ago, he was told to continue his current medica- tions but also started on slow-release niacin because diet, exercise, and other lifestyle modifications, and his current medications, were not ade- quate. What was the most likely reason for adding the niacin?

a. Counteract deficiencies of B-vitamin absorption caused by the antidepressant
b. Counteract polyphagia, and overeating, caused by the metformin
c. Lower triglyceride levels that did not respond adequately to the statin
d. Prevent statin-induced neuropathy
e. Slow the progression of diabetic nephropathy caused by the ACE inhibitor

A

c. Lower triglyceride levels that did not respond adequately to the statin

How well did you know this?
1
Not at all
2
3
4
5
Perfectly
61
Q

A patient presents with severe hypertension and tachycardia. Blood chemistry results, MRI findings, and the overall clinical presentation point to pheochromocytoma. The tumor appears operable, but the patient will have to wait a couple of months for the surgery. We prescribe phenoxy- benzamine in the interim, with the goal of suppressing some of the major signs and symptoms caused by the tumor. Which of the following best summarizes what phenoxybenzamine does, or how it acts?

a. Controls blood pressure by blocking α-adrenergic receptors in the peripheral vasculature
b. Controls heart rate by selectively blocking β1-adrenergic receptors
c. Inhibits catecholamine synthesis in the adrenal (suprarenal) medulla
d. Lowers blood pressure by inhibiting angiotensin converting enzyme and bradykininase
e. Stimulates catechol-O-methyltransferase, thereby facilitating epinephrine’s metabolic inactivation

A

a. Controls blood pressure by blocking α-adrenergic receptors in the peripheral vasculature

How well did you know this?
1
Not at all
2
3
4
5
Perfectly
62
Q
  • **We administer a drug with the intent of lowering a patient’s elevated LDL and total cholesterol levels, and raising HDL levels. The drug we choose inhibits cholesterol synthesis by inhibiting 3-hydroxy-3-methylglutaryl- coenzyme A (better known as [HMG CoA] reductase). Which of the fol- lowing drugs best fits this description and works by the stated mechanism of action?
    a. Clofibrate
    b. Gemfibrozil
    c. Lovastatin
    d. Nicotinic acid (niacin)
    e. Probucol
A

c. Lovastatin

How well did you know this?
1
Not at all
2
3
4
5
Perfectly
63
Q
  • **We are administering nitroprusside intravenously for control of severe hypertension during surgery. The dose has gotten too high, and the drug has been administered too long. Refractoriness to the antihyperten- sive effects has occurred. Blood pressure is rising, and other signs and symptoms of potentially severe toxicity develop. What is the main nitro- prusside metabolite that accounts for these problems?
    a. A highly efficacious α-adrenergic agonist
    b. An extraordinarily potent and irreversible Na-K-ATPase inhibitor
    c. An irreversible agonist for angiotensin II receptors
    d. Cyanide
    e. Nitric oxide
A

d. Cyanide

How well did you know this?
1
Not at all
2
3
4
5
Perfectly
64
Q
  • **We have a patient who is diagnosed with variant (vasospastic) angina. Which of the following drugs would be most appropriate, and gen- erally regarded as most effective, for long-term therapy aimed at reducing the incidence or severity of the coronary vasospasm?
    a. Aspirin
    b. Atorvastatin
    c. Diltiazem
    d. Nitroglycerin
    e. Propranolol
A

c. Diltiazem

How well did you know this?
1
Not at all
2
3
4
5
Perfectly
65
Q

It is generally acceptable and common to administer unfractionated heparin along with other classes of drugs that affect some aspect of the coagulation or thrombolytic processes. The proviso, of course, is to moni- tor closely all drug dosages, the appropriate blood tests, and the patient’s responses overall, since the main risk is uncontrolled or excessive bleeding, if not frank hemorrhage.
There is one main exception. With which one of the following drugs is concomitant administration of heparin contraindicated because of an extremely high risk of excessive bleeding or frank hemorrhage?
a. Alteplase (t-PA)
b. Aspirin
c. Clopidogrel
d. Streptokinase
e. Warfarin

A

d. Streptokinase

SK not clot specific

How well did you know this?
1
Not at all
2
3
4
5
Perfectly
66
Q

A 44-year-old obese man has extremely high plasma triglyceride lev- els, but cholesterol levels are within normal limits. Following treatment with a drug specifically indicated for hypertriglyceridemia, triglyceride levels decrease to almost normal. Which of the following agents is most likely to have caused this desired change?

a. Atorvastatin
b. Cholestyramine
c. Colestipol
d. Ezitemibe
e. Gemfibrozil

A

e. Gemfibrozil

PPAR activator, stimulates lipoprotein lipase synsthesis

How well did you know this?
1
Not at all
2
3
4
5
Perfectly
67
Q

A patient has received excessive doses of nitroprusside, and toxic manifestations are developing. Which of the following drugs should we administer to help nitroprusside’s metabolism proceed to the formation of a less toxic metabolite?

a. Epinephrine
b. Sodium thiosulfate
c. Thrombin
d. Vitamin C
e. Vitamin E
f. Vitamin K

A

b. Sodium thiosulfate

68
Q

Nicotinic acid (niacin), in large doses that are used to treat hyper triglyceridemia, causes an often-disturbing cutaneous flush. Which one of the following mechanisms most likely contributes to the vasodilatory response and the flushing?

a. Activation of α-adrenergic receptors
b. Calcium channel activation in vascular smooth muscle
c. Production of local prostaglandins
d. Release of angiotensin II
e. Release of histamine

A

c. Production of local prostaglandins (give ASA to prevent)

69
Q

A 45-year-old man postmyocardial infarction (MI) is being treated with several drugs, including intravenous unfractionated heparin. Stool guaiac on admission was negative, but is now four, and he has had an episode of hematemesis. Which of the following would be the best drug to administer to counteract the effects of excessive heparin remaining in the circulation?

a. Aminocaproic acid
b. Dipyridamole
c. Factor IX
d. Protamine sulfate
e. Vitamin K

A

d. Protamine sulfate

70
Q
  • **A 45-year-old man asks his physician for a prescription for sildenafil to improve his sexual performance. Because of risks from a serious drug interaction, this drug should not be prescribed, and the patient should be urged not to try to obtain it from other sources, if he is also taking which of the following drugs?
    a. An angiotensin-converting enzyme inhibitor
    b. A β-adrenergic blocker
    c. A nitrovasodilator (e.g., nitroglycerin)
    d. A statin-type antihypercholesterolemic drug
    e. A thiazide or loop diuretic
A

c. A nitrovasodilator (e.g., nitroglycerin)

“do not take this drug if you are taking nitrates for cp”

71
Q

A physician is preparing to administer a drug for which there is a label warning: “do not administer this drug to patients with second-degree or greater heart block, or give with other drugs that may cause heart block.” Which of the following findings would be specifically indicative of heart block and second-degree heart block in particular?
a. Auscultation of the precordium reveals an irregular rhythm
b. Blood pressure is low
c. Heart rate is abnormally low (bradycardia), but there is normal sinus rhythm
d. The EKG reveals ventricular ectopic beats
e. The EKG shows a prolonged PR interval, and some P waves are not followed by
a normal QRS complex
f. The EKG shows abnormally widened QRS complexes

A

e. The EKG shows a prolonged PR interval, and some P waves are not followed by
a normal QRS complex

72
Q
  • **A 70-year-old woman is treated with sublingual nitroglycerin for occasional bouts of effort-induced angina. Which of the following best describes the mechanism by which nitroglycerin causes its desired antiang- inal effects, or a mediator involved in it?
    a. Blocks α-adrenergic receptors
    b. Forms cyanide, much like the metabolism of nitroprusside does
    c. Increases local synthesis and release of adenosine
    d. Raises intracellular cGMP levels
    e. Stimulates phosphodiesterase
A

d. Raises intracellular cGMP levels

73
Q

A 67-year-old patient complains of muscle aches, pain, and tenderness. These affect the legs and trunk. There is no fever, bruising, or any recent his- tory of muscle trauma or strains (as from excessive exercise). There is myo- globinuria, a clinically significant fall of creatinine clearance, and a rise of serum creatine kinase (CK) to levels nearly 10 times the upper limit of nor- mal. Which of the following drugs is the most likely cause of these findings?

a. Aspirin (“low dose”) for its cardioprotective/antiplatelet effects
b. Captopril for hypertension and heart failure
c. Carvedilol for hypertension, heart failure, and angina prophylaxis
d. Furosemide as adjunctive management of his heart failure
e. Nitroglycerin, sublingual, for effort-induced angina
f. Pravastatin to control his hypercholesterolemia and the associated risks

A

f. Pravastatin to control his hypercholesterolemia and the associated risks

74
Q

A 20-year-old varsity hockey player is referred to you by his coach. The young athlete has excessive bruising after a very physical match 2 days before. His knee had been bothering him, so he took two 325-mg aspirin tablets several hours before the contest. He got checked hard into the boards 10 times during the game, but denies any excessive or unusual trauma. As you ponder the etiology you order several blood tests. Which test or finding do you most likely expect to be abnormal as a result of the prior aspirin use?

a. Activated partial thromboplastin time (APTT)
b. Bleeding time
c. INR (International Normalized Ratio)
d. Platelet count
e. Prothrombin time

A

b. Bleeding time

75
Q

A 60-year-old man, hospitalized for an acute myocardial infarction, is treated with warfarin (among other drugs). What is the main mechanism by which warfarin is causing the effects for which it is given?

a. Increase in the plasma level of Factor IX
b. Inhibition of thrombin and early coagulation steps
c. Inhibition of synthesis of prothrombin and coagulation Factors VII, IX, and X
d. Inhibition of platelet aggregation in vitro
e. Activation of plasminogen
f. Binding of Ca2+ ion cofactor in some coagulation steps

A

c. Inhibition of synthesis of prothrombin and coagulation Factors VII, IX, and X

76
Q

A 30-year-old pregnant woman requires heparin for prophylaxis of thromboembolism. Which of the following best summarizes heparin’s main mechanism of action?

a. Activates plasminogen
b. Increases the plasma level of Factor IX
c. Inhibits platelet aggregation in vitro
d. Inhibits synthesis of prothrombin and coagulation Factors VII, IX, and X
e. Inhibits thrombin and early coagulation steps
f. Lyses platelets

A

e. Inhibits thrombin and early coagulation steps

77
Q

A 42-year-old man with an acute MI is treated with alteplase. Which of the following most accurately describes how this drug exerts its intended effect?

a. Blocks platelet ADP receptors
b. Inhibits platelet thromboxane production
c. Inhibits synthesis of vitamin K-dependent coagulation factors
d. Prevents aggregation of adjacent platelets by blocking Glycoprotein IIb/IIIa receptors
e. Promotes conversion of plasminogen to plasmin

A

e. Promotes conversion of plasminogen to plasmin

78
Q

A 64-year-old woman has had several episodes of transient ischemic attacks (TIAs). Aspirin would be a preferred treatment, but she has a his- tory of severe “aspirin sensitivity” manifest as intense bronchoconstriction and urticaria. Which of the following would you consider to be the best alternative to the aspirin?

a. Acetaminophen
b. Aminocaproic acid
c. Clopidogrel
d. Dipyridamole
e. Streptokinase

A

c. Clopidogrel

79
Q

A 60-year-old woman with deep-vein thrombosis (DVT) is given a bolus of heparin, and a heparin drip is also started. Thirty minutes later she is bleeding profusely from the intravenous site. The heparin is stopped, but the excessive bleeding continues. You decide to give protamine sulfate to reverse the adverse effect of heparin. Which statement best describes the mechanism of action of this antidote?

a. Activates the coagulation cascade, overriding the action of heparin
b. Causes hydrolytic inactivation of heparin
c. Causes platelet aggregation, thereby providing a natural hemostatic effect
d. Changes the conformation of antithrombin III to prevent binding to heparin
e. Combines with heparin as an ion pair, thus neutralizing it

A

e. Combines with heparin as an ion pair, thus neutralizing it

80
Q

A patient has been receiving otherwise “proper” doses of a drug for 5 days straight. Dosing was done correctly, starting with usual maintenance doses; no loading dose strategy was used. Then, and rather precipitously, they develop signs and symptoms of widespread thrombotic events; platelet counts decline significantly concomitant with the thrombosis. The patient dies within 24 h of the onset of signs and symptoms. Which is the most likely cause?

a. Abciximab
b. Clopidogrel
c. Heparin (unfractionated)
d. Nifedipine
e. Warfarin

A

c. Heparin (unfractionated)

heparin-induced thrombocytopenia - HIT - It is an immune-mediated thrombocytopenia

81
Q
  • **A patient on long-term warfarin therapy arrives at the clinic for her weekly prothrombin time measurement. Her INR is dangerously pro- longed, and the physical exam reveals petechial hemorrhages. She’s had episodes of epistaxis over the last 2 days. We are going to stop the warfarin until the INR becomes acceptable (and perhaps admit the patient for follow-up). However, we are concerned with her ongoing bleeding. Which of the following drugs would you most likely administer to counteract the warfarin’s excessive effects?
    a. Aminocaproic acid
    b. Epoetin alfa
    c. Ferrous sulfate
    d. Phytonadione (vitamin K)
    e. Protamine sulfate
A

d. Phytonadione (vitamin K)

82
Q

A 58-year-old man presents with a myocardial infarction—his first episode of ACS. Angioplasty and stenting are not possible because the car- diac cath lab is busy with other high-priority patients, so administration of a thrombolytic drug is the only option. Which one of the following is the most important determinant, overall, of the success of thrombolytic ther- apy in terms of salvaging viable cardiac muscle?

a. Choosing a “human” (cloned) plasminogen activator (e.g., t-PA), rather than one that is bacterial-derived
b. Infarct location (i.e., anterior wall of left ventricular vs. another site/wall)
c. Presence of collateral blood vessels to the infarct-related coronary artery
d. Systolic blood pressure at the time the MI is diagnosed
e. Time from onset of infarction to administration of the thrombolytic agent

A

e. Time from onset of infarction to administration of the thrombolytic agent

83
Q
  • **A patient with coronary artery disease takes excessive doses of sublin- gual nitroglycerin in an attempt to abort chest discomfort from an ischemic episode. By the time help arrives he is dead from an overdose of the very drug he took to seek relief. Which of the following best describes the most likely mechanism by which this antianginal/anti-ischemic drug led to the patient’s death?
    a. Caused fatal bronchospasm
    b. Caused generalized convulsive seizures, with death arising from apnea
    c. Caused hemorrhagic stroke from hypertension
    d. Provoked coronary vasospasm
    e. Worsened myocardial ischemia
A

e. Worsened myocardial ischemia

84
Q

patient is started on therapy with abciximab. Which one of the following best describes how this drug causes its desired effects?

a. Blocks thrombin receptors selectively
b. Blocks ADP receptors
c. Blocks glycoprotein IIb/IIIa receptor
d. Inhibits cyclooxygenase
e. Inhibits prostacyclin production

A

c. Blocks glycoprotein IIb/IIIa receptor

85
Q

***A patient with angina is started on a nitroglycerin transdermal deliv- ery system (“skin patch”) for prophylaxis of his angina. He wears the patch 24 h a day, 7 days a week, except for the few minutes when he showers each day. Which of the following is the main concern with “around-the- clock” administration of this or other long-acting formulations of nitrova- sodilators?
a. Cyanide poisoning
b. Development of tolerance to their vasodilator actions
c. Gradual development of reflex bradycardia in response to successive doses
d. Onset of delayed, characteristic adverse responses including thrombosis and
thrombocytopenia
e. Paradoxical vasoconstriction leading to hypertension

A

b. Development of tolerance to their vasodilator actions

86
Q

Many clinical studies have been conducted to investigate the benefits of daily aspirin use in the primary prevention of coronary heart disease and sudden death in adults. The results have been somewhat inconsistent, in part because different dosages were studied, and there were important dif- ferences in the populations that were studied. Nonetheless, many (if not most) of the studies have revealed that for some at-risk patients, aspirin increased the incidence of a particularly unwanted adverse response, even when dosages were kept within the range of dosages typically recom- mended for cardioprotection (81 mg/day). Which of the following is the most likely adverse response associated with the drug?

a. Centrolobular hepatic necrosis
b. Hemorrhagic stroke
c. Nephropathy
d. Tachycardia and hypotension leading to acute myocardial ischemia
e. Vasospastic angina

A

b. Hemorrhagic stroke

87
Q
  • **A 59-year-old man presents in the emergency department with crushing chest discomfort. An EKG indicates a transmural infarction, and prompt cardiac catheterization and assessment of prior lab results indicate significant hypercholesterolemia. The patient is given all the drugs listed, for both immediate management of the ischemia and its symptoms and for long-term prevention of a subsequent, and potentially fatal, MI. Which one of the following drugs provides immediate relief of the consequences of myocardial ischemia but has no long-term effects to reduce the risk of sud- den death or ventricular dysfunction from another MI?
    a. Aspirin
    b. Atorvastatin
    c. Captopril
    d. Nitroglycerin
    e. Propranolol
A

d. Nitroglycerin

88
Q

A 50-year-old man is well aware of the benefits of aspirin in terms of reducing the risk of death from an acute myocardial infarction, mainly because he has seen and carefully studied many of the ads and internet posts about this. He notices that the usual recommended dose of aspirin for cardioprotection is 81 mg/day, but reasons that the bigger the dose, the big- ger the protective effect. He has taken “at least” 1000 mg of aspirin twice a day for the last 6 months. While he is fortunate in terms of having no apparent gastrointestinal adverse effects, he suffers an MI. Autopsy results show considerable platelet occlusion of several coronary vessels. Which of the following most likely explains the mechanism by which aspirin trig- gered these events?

a. Acetylated platelet glycoprotein IIb/IIIa receptors, triggering aggregation
b. Favored adhesion of platelets to the vascular (coronary) endothelium
c. Ruptured atherosclerotic plaque in the coronaries, exposing platelets to collagen
d. Suppressed hepatic synthesis of vitamin K-dependent clotting factors
e. Triggered excessive activation of platelets by ADP

A

b. Favored adhesion of platelets to the vascular (coronary) endothelium

(at higher doses, aspirin also inhibits synthesis of other eicosanoids, of which PGI2, synthesized in the vascular endothelium, is of most importance here. Endothelial prostacyclin synthesis helps prevent platelets from adhering to the vascular wall Suppress PGI2 in the endothelium, and platelet adherence is increased, despite the fact that platelet TXA2 synthesis has already been blocked)

89
Q

The figure below shows typical cardiovascular responses to the intravenous
injection of four adrenergic drugs into a normal, resting subject.
Assume the doses of each are sufficient to cause expected effects, but not so
high that toxic effects occur. No other drugs are present, and sufficient time has
been allowed to enable complete dissipation of the effects of any prior drugs,
and a return to normal resting hemodynamic status. The dashed line between
the systolic and diastolic pressure traces represents mean arterial pressure.
Which of the following letter answers indicates the drugs that are
ordered in the sequence shown (1, 2, 3, 4)?
a. EPI, NE, PHE, ISO
b. ISO, EPI, NE, PHE
c. ISO, PHE, EPI, NE
d. NE, ISO, PHE, EPI
e. PHE, EPI, NE, PRO
f. PHE, ISO, NE, EPI
g. PRO, PHN, PHE, ISO

A

c. ISO, PHE, EPI, NE

90
Q

A 65-year-old man with severe congestive heart failure (CHF) is unable to climb a flight of stairs without experiencing shortness of breath. After several
years of therapy with first-line drugs for heart failure, we empirically try digoxin to improve cardiac muscle contractility. Within 4 weeks, he has a marked improvement in his symptoms. Which of the following best describes the main cellular action of digoxin that accounts for its ability to improve his overall wellness and his cardiovascular function in particular?
a. Activates beta1-adrenergic receptors
b. Facilitates GTP binding to specific G proteins
c. Increases mitochondrial calcium (Ca2+) release
d. Inhibits sarcolemmal Na+-K+-ATPase
e. Stimulates cyclic adenosine 5’-monophosphate (cAMP) synthesis

A

d. Inhibits sarcolemmal Na+-K+-ATPase

91
Q
  • ***A patient has periodic episodes of paroxysmal supraventricular tachycardia (PSVT). Which of the following drugs would be most suitable for outpatient prophylaxis of these worrisome electrophysiologic events?
    a. Adenosine
    b. Lidocaine
    c. Nifedipine
    d. Nitroglycerin
    e. Verapamil
A

e. Verapamil

92
Q

We prescribe a beta-adrenergic blocker for a patient with chronic-stable (“effort-induced”) angina, and the incidence and severity of anginal attacks
are reduced. Which of the following best explains the pharmacologic action by which the beta blocker does this?
a. Decreases myocardial oxygen demand
b. Dilates the coronary vasculature
c. Exerts antiplatelet/antithrombotic effects
d. Reduces total peripheral resistance
e. Slows AV nodal conduction velocity

A

a. Decreases myocardial oxygen demand

93
Q

Your patient is a 50-year-old man with well-controlled Type 2 diabetes and normal renal function (and no microalbuminuria). Which of the following drugs would be the most rational first choice for starting his antihypertensive therapy?

a. Angiotensin-converting enzyme (ACE) inhibitor or angiotensin receptor blocker
b. beta-adrenergic blocker
c. Nifedipine
d. Thiazide diuretic
e. Verapamil or diltiazem

A

a. Angiotensin-converting enzyme (ACE) inhibitor or angiotensin receptor blocker

(ACE inhibitors are generally the preferred drugs for hypertensive patients who also have diabetes mellitus provided their renal function is satisfactory)

94
Q
We have a 50-year-old man with asymptomatic hyperuricemia, and we are about to start therapy for newly diagnosed essential hypertension (BP 136/90 mm Hg, based on repeated measurements with the patient supine and at rest). Which of the following antihypertensive drugs is most likely to increase his serum uric acid levels further, and possibly precipitate
a gout attack?
a. Captopril
b. Hydrochlorothiazide
c. Labetalol
d. Losartan
e. Verapamil
A

b. Hydrochlorothiazide

95
Q
We've just diagnosed essential hypertension in a 58-year-old female patient. She tends to be tachycardic. Notes written by her ophthalmologist
indicate that she has chronic open-angle glaucoma. Which of the following drugs would be the most rational choice for this woman, given only the
information presented in this question?
a. Captopril
b. Diltiazem
c. Hydrochlorothiazide
d. Timolol
e. Verapamil
A

d. Timolol

96
Q

*Our newly diagnosed hypertensive patient has a history of vasospastic angina. Which of the following drugs or drug classes would be the most rational for starting antihypertensive therapy because it exerts antihypertensive effects, directly lowers myocardial oxygen demand and consumption, and also tends to inhibit cellular processes that otherwise favor
coronary vasospasm?
a. Angiotensin-converting enzyme (ACE) inhibitor or angiotensin receptor blocker
b. beta-Adrenergic blocker
c. Nifedipine
d. Thiazide diuretic
e. Verapamil (or diltiazem)

A

e. Verapamil (or diltiazem)

97
Q
  • **We have a patient with newly diagnosed essential hypertension, and start them on a commonly used antihypertensive drug at a dose that is considered to be therapeutic for the vast majority of patients. Soon after starting therapy the patient experiences crushing chest discomfort. EKG changes show myocardial ischemia. Studies in the cardiac cath lab show episodes of coronary vasospasm, and it is likely the antihypertensive drug provoked the vasoconstriction. Which of the following antihypertensive drugs or drug class most likely caused the ischemia and the angina?
    a. Atenolol
    b. Diltiazem
    c. Hydrochlorothiazide
    d. Losartan
    e. Metolazone
A

a. Atenolol (NO Betal blockers in vasospastic angina)

98
Q

***A 28-year-old woman is receiving drug therapy for essential hypertension. She subsequently becomes pregnant. We realize that the drug she’s been taking for her high blood pressure can have serious, if not fatal, effects on the fetus (it is in pregnancy category X). As a result, we stop the drug and substitute another that is deemed to be equally efficacious in terms of
her blood pressure, and safer for the fetus. Which of the following drugs was she most likely taking before she became pregnant?
a. altha-Methyldopa
b. Captopril
c. Furosemide
d. Labetalol
e. Verapamil

A

b. Captopril

ACE inhibitors and ARBs contraindicated in pregnancy

99
Q

***We have just diagnosed Stage 1 essential hypertension in a 30-year old
man who has a history of asthma. He regularly uses an inhaled corticosteroid,
which seems to work well, but does need to use an albuterol inhaler about once every 3 weeks for suppression of asthma attacks. Which antihypertensive drug or drug class poses the greatest risk of exacerbating the patient’s asthma and counteracting the desired pulmonary effects of the albuterol, even though it might control his blood pressure well?
a. Diltiazem
b. Hydrochlorothiazide
c. Labetalol
d. Ramipril
e. Verapamil

A

C. Labetalol

no BB in asthma

100
Q

A healthy adult subject is given an intravenous injection of a test drug. Both blood pressure and total peripheral resistance rise promptly. This is followed immediately by a reduction of heart rate. In repeated experiments we find that the vasopressor response is not affected by pretreatment with prazosin. However, pretreatment with atropine prevents the cardiac chronotropic response. The test drug was most likely which of the following?

a. Angiotensin II
b. Dobutamine
c. Isoproterenol
d. Norepinephrine
e. Phenylephrine

A

a. Angiotensin II

the bradycardia is reflex blocked by atropine

101
Q

Quinidine is ordered for a patient with recurrent atrial fibrillation and who refuses any interventions other than drugs in an attempt to terminate and control the arrhythmia. Which of the following is the most likely effect of quinidine?

a. Is likely to increase blood pressure via a direct vasoconstrictor effect
b. Is contraindicated if the patient also requires anticoagulant therapy
c. Slows spontaneous SA nodal depolarization as its predominant effect
d. Tends to slow electrical impulse conduction velocity through the AV node
e. Will increase cardiac contractility (positive inotropic effect) independent of its antiarrhythmic effects

A

c. Slows spontaneous SA nodal depolarization as its predominant effect

(Quinidine’s main beneficial effect in supraventricular arrhythmias
is a suppression of spontaneous depolarization of the SA node with concomitant, effect on the AV node is an increase of nodal electrical impulse conduction velocity)

102
Q
A patient who has been taking an oral antihypertensive drug for
about a year develops a positive Coombs' test. Which of the following
drugs is the most likely cause?
a. Captopril
b. Clonidine
c. Labetalol
d. Methyldopa
e. Prazosin
A

d. Methyldopa

103
Q

We want to compare and contrast thec ardiac and hemodynamic profiles of immediateacting dihydropyridine-type calcium channel blockers (CCBs) and the nondihydropyridines, verapamil, and diltiazem (benzothiazepines). Which of the following best summarizes how verapamil or diltiazem differs from nifedipine?

a. Are suitable for use in conjunction with a β blocker or digoxin
b. Cause a much higher incidence of reflex tachycardia
c. Cause significant dose-dependent slowing of AV nodal conduction velocity
d. Cause significant venodilation, leading to profound orthostatic hypotension
e. Have significant positive inotropic effects

A

c. Cause significant dose-dependent slowing of AV nodal conduction velocity

104
Q

Digoxin affects a host of cardiac electrophysiologic properties. Some of its effects are caused directly by the drug. Others are indirect: they may involve increasing “vagal tone” to the heart or other compensations that arise when cardiac output is improved in a patient with heart failure. For some parameters the direct and indirect effects may be qualitatively (but not quantitatively) opposing, but one will predominate over the other. Which of the following is an expected and predominant effect of the drug?

a. Increased rate of SA nodal depolarization
b. Reduced atrial automaticity
c. Reduced ventricular automaticity
d. Slowed AV nodal conduction velocity
e. Slowed conduction velocity through the atrial myocardium and His-Purkinje system

A

d. Slowed AV nodal conduction velocity

105
Q
  • **A patient has Stage III essential hypertension. After evaluating the responses to several other antihypertensive drugs, alone and in combina- tion, the physician places the patient on oral hydralazine. Which of the following adjunct(s) is/are likely to be needed to manage the expected and unwanted cardiovascular side effects of the hydralazine?
    a. Captopril plus nifedipine
    b. Digoxin plus spironolactone
    c. Digoxin plus vitamin K
    d. Hydrochlorothiazide and a β blocker
    e. Nitroglycerin
    f. Triamterene plus amiloride
A

d. Hydrochlorothiazide and a β blocker

(Hydralazine predominately dilates arterioles, with negligible effects on venous capacitance It typically lowers blood pressure “so well” that it can trigger reflex tachycardia and RAAS)

106
Q

We have a patient who is taking a tricyclic antidepressant (e.g., imipramine), which blocks neuronal catecholamine reuptake. We now have to start him on antihypertensive drug therapy. If we continue the tri- cyclic, which of the following drugs will most likely have its antihypertensive effects inhibited?

a. Diazoxide
b. Guanadrel
c. Hydralazine
d. Prazosin
e. Propranolol

A

b. Guanadrel

(Guanethidine is occasionally used for severe or refractory hypertension. It must be taken up by adrenergic nerve endings in order for it to exert its effects)

107
Q
  • **A 64-year-old man with coronary atherosclerosis and “mild” heart failure has been treated with digoxin and several other drugs. He com- plains of nausea, vomiting, and diarrhea. His EKG reveals a bigeminal rhythm and second-degree heart block. A drug-drug interaction is suspected. Which of the following coadministered drugs most likely provoked the problem?
    a. Captopril
    b. Cholestyramine
    c. Furosemide
    d. Lovastatin
    e. Nitroglycerin
A

c. Furosemide

(Hypokalemia due to the effects of potassium- wasting diuretics such as furosemide increase susceptibility to digoxin tox- icity, and they are probably the most common cause of it)

108
Q

At high (but not necessarily toxic) blood levels,a cardiovascular drug with which you should be familiar causes many signs and symptoms that resemble what you see with “low-grade” aspirin toxicity (salicylism): light-headedness, tinnitus, and visual disturbances such as diplopia.
What is the most likely drug that caused these responses?
a. Atropine
b. Captopril
c. Dobutamine
d. Propranolol
e. Quinidine

A

e. Quinidine

109
Q

A 56-year-old man has heart failure. His family doctor, who has been caring for him since he was a young lad, has been treating him with digoxin, furosemide, and triamterene for several years. The patient now develops atrial fibrillation, and so his doctor starts quinidine and warfarin. Which of the following is the most likely outcome of adding the quinidine?

a. Development of signs and symptoms of quinidine toxicity (cinchonism)
b. Hyponatremia due to quinidine’s ability to enhance diuretic-induced sodium loss
c. Onset of signs and symptoms of digoxin toxicity
d. Precipitous development of hypokalemia
e. Prompt suppression of cardiac contractility, onset of acute heart failure

A

c. Onset of signs and symptoms of digoxin toxicity

110
Q

You pass a room where a cardiologist is talking about various antiarrhythmic drugs. You stop and listen just as he comments on antiarrhythmics in Vaughan-Williams Class I-c. Which of the following is the most likely correct and clinically relevant “take home” message about antiar- rhythmics in this class?
a. Are only given for arrhythmias during acute myocardial infarction
b. Are particularly suited for patients with low ejection fractions
c. Are preferred drugs (drugs of choice) for relatively innocuous ventricular
arrhythmias
d. Cause pulmonary fibrosis and a hypothyroid-like syndrome when given long
term
e. Have a significant pro-arrhythmic effect (induction of lethal arrhythmias)

A

e. Have a significant pro-arrhythmic effect (induction of lethal arrhythmias)

111
Q

A patient with Stage 2 essential hypertension is treated with usually effective doses of an ACE inhibitor. After a suitable period of time blood pressure has not been lowered satisfactorily. The patient has been compliant with drug therapy and other recommendations (e.g., weight reduction, exercise). A thiazide is added to the ACE inhibitor regimen. Which of the following is the most likely and earliest untoward outcome of this drug add-on, for which you should monitor closely?
a. Fall of blood pressure sufficient to cause syncope
b. Hypokalemia due to synergistic effects of the ACE inhibitor and the thiazide on
renal potassium excretion
c. Onset of acute heart failure from depression of ventricular contractility
d. Paradoxical hypertensive crisis
e. Sudden prolongation of the P-R interval and increasing degrees of heart block

A

a. Fall of blood pressure sufficient to cause syncope

112
Q
  • **A 52-year-old woman with essential hypertension, hypercholesterolemia, and chronic-stable angina develops severe constipation. It is attributed to one of her medications. Which was the most likely cause?
    a. Atorvastatin
    b. Captopril
    c. Labetalol
    d. Nitroglycerin
    e. Verapamil
A

e. Verapamil

113
Q

Angiotensin-converting enzyme (ACE) inhibitors are among the first-line drugs for managing essential hypertension (prehypertension or Stage 1 hypertension). However, in contrast with the main alternatives (thiazide or thiazide-like diuretics, beta blockers, or calcium channel blockers), the ACE inhibitors are associated with a comparatively high incidence of a rather unusual adverse reaction. Which of the following is that?

a. Bradycardia, often involving AV block
b. Hepatitis
c. Hirsutism
d. Hypokalemia
e. Proteinuria, renal insufficiency

A

e. Proteinuria, renal insufficiency

One of the adverse responses to ACE inhibitors is impairment of renal function, as evidenced by proteinuria

114
Q
  • **A patient with multiple cardiovascular disorders present in your clinic. His primary complaints are fever and arthralgia, and other “flu-like symptoms.” These findings, plus a facial “rash” and results of blood work, all point to a drug-induced lupus-like syndrome. Heart rate, BP, and all other cardiovascular findings are completely normal. Which of the following drugs is the most likely cause of these findings?
    a. Aspirin (“low dose”) for its cardioprotective/antiplatelet effects
    b. Atorvastatin for primary prevention of CAD
    c. Captopril for hypertension and heart failure
    d. Carvedilol for hypertension, heart failure, and angina prophylaxis
    e. Furosemide as adjunctive management of his heart failure
    f. Nitroglycerin, sublingual, for effort-induced angina
    g. Procainamide for atrial fibrillation
A

g. Procainamide for atrial fibrillation

115
Q

We use invasive hemodynamic techniques to measure or calculate the effects of various drugs on such parameters as arterial pressure, total peripheral resistance, and central venous (right atrial) pressures. Our goal is to evaluate whether the drug primarily causes arteriolar or venular dilation, or affects both sides of the circulation. Which one of the following drugs exerts vasodilator effects in both the arterial and venous circulations?

a. Diazoxide
b. Hydralazine
c. Minoxidil
d. Nitroprusside
e. Nifedipine

A

d. Nitroprusside

(Nitroprusside relaxes both arterioles and venules Diazoxide, hydralazine, minoxidil, nifedipine relax arteriolar smooth muscle more)

116
Q

A patient in the coronary care unit develops episodes of paroxysmal AV nodal reentrant tachycardia (PSVT). Which of the following would generally be considered a first-line drug for promptly stopping the arrhythmia?

a. Adenosine
b. Digoxin
c. Edrophonium
d. Phenylephrine
e. Propranolol

A

a. Adenosine

117
Q
  • *An 83-year-old man has been effectively treated with hydrochlorothiazide to control his elevated blood pressure. He has had a recent onset of weakness. Blood chemistry analysis reveals hypokalemia. Another drug is added, and 1 month later his serum K+ is normal. Which of the following drugs most likely helped normalize his serum potassium levels?
    a. Acetazolamide
    b. Amiloride
    c. Furosemide
    d. Metolazone
    e. Mannitol
A

b. Amiloride

(Amiloride is a K-sparing diuretic with a mild diuretic and
natriuretic effect)

118
Q
A 66-year-old woman with heart failure and mild hearing loss is given a diuretic as part of a regimen that includes a beta blocker and an ACE inhibitor. In the course of treatment she develops an AV conduction defect and is found to be hypocalcemic and hypomagnesemic. She also reports what she describes as some worsening of her hearing, characterized mainly by an inability to distinguish between similarly sounding words. Audiometry
confirms the hearing loss. The signs and symptoms noted here abate when the drug is stopped. Which of the following drugs most likely contributed
to these findings?
a. Acetazolamide
b. Amiloride
c. Furosemide
d. Hydrochlorothiazide
e. Mannitol
A

c. Furosemide

119
Q

A patient with atrial fibrillation is placed on long-term arrhythmia control with amiodarone. In addition to “standard” monitoring, periodic assessments of which of the following should be made in order to detect adverse effects that are unique to this drug?

a. Blood glucose, triglyceride, cholesterol, and sodium concentrations
b. Hearing thresholds (audiometry) and serum albumin concentration
c. Prothrombin time and antinuclear antibody (ANA) titers
d. Pulmonary function and thyroid hormone status
e. White cell counts and serum urate concentration

A

d. Pulmonary function and thyroid hormone status

Pulmonary fibrosis has been reported with long-term amiodarone therapy

120
Q
  • **A patient, who has excessively slow AV nodal conduction rates that unfortunately haven’t been recognized, is started on a drug. As soon as blood levels climb towards the usual therapeutic range the patient goes into complete heart block. Which of the following drugs most likely provoked this further prolongation of the P-R interval, ultimately leading to the complete heart block?
    a. Captopril
    b. Losartan
    c. Nifedipine
    d. Nitroglycerin
    e. Prazosin
    f. Verapamil
A

f. Verapamil

121
Q

A patient with heart failure, Stage 2 essential hypertension, and hyperlipidemia (elevated LDL cholesterol and abnormally low HDL-C) is taking furosemide, captopril, atenolol, and simvastatin (an HMG CoA reductase inhibitor).
During a scheduled physical exam, about a month after starting all the above drugs, the patient reports a severe, hacking, and relentless cough. Other vital signs, and the overall physical assessment, are consistent with good control of both the heart failure and blood pressure and indicate no other underlying disease or abnormalities. Results of blood tests are not yet available.
Which of the following is the most likely cause of the cough?
a. An expected side effect of the captopril
b. An allergic reaction to the statin
c. Dyspnea due to captopril’s known and powerful bronchoconstrictor action
d. Excessive doses of the bumetanide, which led to hypovolemia
e. Hyperkalemia caused by an interaction between bumetanide and captopril
f. Pulmonary edema from the bumetanide

A

a. An expected side effect of the captopril

122
Q

*A patient with a history of hypertension, heart failure, and peripheral vascular disease has been on oral therapy with drugs suitable for each for about 3 months. He runs out of the medication and plans to have the prescriptions refilled in a week or so.
Within a day or two after stopping his medications he experiences an episode of severe tachycardia accompanied by tachyarrhythmias, and an abrupt rise of blood pressure to 240/140 mm Hg—well above pretreatment levels. He complains of chest pain, anxiety, and a pounding headache. Soon thereafter he suffers a hemorrhagic stroke.
Which of the following drugs or drug groups, the man suddenly stops taking, most likely causes these responses?
a. ACE inhibitors
b. Clonidine
c. Digoxin
d. Furosemide
e. Nifedipine (a long-acting formulation)
f. Warfarin

A

b. Clonidine

123
Q

***Your patient has bipolar illness, hypercholesterolemia, chronic-stable angina, and Stage I essential hypertension. He has been taking lithium and an SSRI for the bipolar illness. Cardiovascular drugs include atorvastatin, diltiazem, sublingual nitroglycerin, captopril, and hydrochlorothiazide. Which of the following outcomes, due to interactions involving these drugs, would you most likely expect?
a. Development of acute psychosis from an ACE inhibitor-antipsychotic
interaction
b. Development of a hypomanic state from antagonism of lithium’s action by the
nitroglycerin
c. Lithium toxicity because of hyponatremia caused by the hydrochlorothiazide
d. Loss of cholesterol control from antagonism of the HMG CoA reductase inhibitor by the antipsychotic
e. Worsening of angina because the antipsychotic counteracts the effects of the calcium channel blocker
f. Worsening of angina because the lithium antagonizes the effects of the
nitroglycerin

A

c. Lithium toxicity because of hyponatremia caused by the hydrochlorothiazide

124
Q

***A patient is hospitalized and waiting for coronary angiography. His history includes angina pectoris that is brought on by “modest” exercise, and is accompanied by transient electrocardiographic changes consistent with myocardial ischemia. There is no evidence of coronary vasospasm. In
the hospital he is receiving nitroglycerin and morphine (slow intravenous infusions), plus oxygen via nasal cannula.
He suddenly develops episodes of chest discomfort. Heart rate during these episodes rises to 170-190 beats/min; blood pressure reaches 180-200/110-120 mm Hg, and prominent findings on the EKG are runs of ventricular ectopic beats that terminate spontaneously, plus ST-segment elevation.
Although there are several things that need to be done for immediate care, administration of which one of the following is most likely to remedy (at least temporarily) the majority of these signs and symptoms and pose the lowest risk of doing further harm?
a. Aspirin
b. Captopril
c. Furosemide
d. Labetalol
e. Lidocaine
f. Nitroglycerin (increased dose as a bolus)
g. Prazosin

A

d. Labetalol

(Given its combination of both α/β blocking effect, it is the best approach for managing the hypertension, the tachycardia, oxygen supply-demand imbalance that leads to both CP and the ischemic ST-changes, and the ventricular ectopy - which is probably a reflection of excessive catecholamine stimulation of β1
receptors)

125
Q

***A first-year house officer notices that a patient is experiencing significant and rapidly rising blood pressure (currently 180/120 mm Hg). One of the medications the patient had been taking is immediate-acting nifedipine oral capsules. There is a dose of this nifedipine formulation at the bedside, so the MD pricks the capsule open and squirts the contents into the
patient’s mouth. This technique avoids “first-pass” metabolism of the drug
and causes rapid absorption and all the effects associated with this calcium
channel blocker. Which of the following is the most likely outcome, given
the scenario?
a. AV nodal block
b. Further rise of heart rate, worsening of the ventricular arrhythmia
c. Hypotension and bradycardia
d. Normalization of blood pressure and heart rate
e. Return of blood pressure toward normal, no significant effect on heart rate or
the EKG

A

b. Further rise of heart rate, worsening of the ventricular arrhythmia

(fast hypotension - baroreceptor reflex)

126
Q
  • *A patient with multiple cardiovascular diseases is being treated with digoxin, furosemide, triamterene, atorvastatin, and nitroglycerin—all prescribed by the family physician he’s had for decades. The patient now experiences nausea, vomiting, and anorexia, and describes a “yellowish-greenish tint” to white objects and bright lights. These signs and symptoms are most characteristic of toxicity due to which one of the following drugs?
    a. Atorvastatin
    b. Digoxin
    c. Furosemide
    d. Nitroglycerin
    e. Triamterene
    f. The triamterene-furosemide combination (drug interaction)
A

b. Digoxin

127
Q

***Your patient has severe (Stage 4) hypertension that is being controlled with a combination of hydralazine, furosemide, and carvedilol. He also has had bouts of atrial fibrillation that are being managed long term with quinidine and warfarin for prophylaxis of thromboembolism. He presents with fever, chills, arthralgia, and a purplish discoloration on the face.
The diagnosis is a drug-induced lupus-like syndrome. Which drug in the regimen noted above is most likely to have accounted for this finding?
a. Carvedilol
b. Furosemide
c. Hydralazine
d. Quinidine
e. Warfarin

A

c. Hydralazine

128
Q
  • **We have a 28-year-old female patient with Stage II essential hypertension, tachycardia, and occasional palpitations (ventricular ectopic beats). Normally we might consider prescribing a beta blocker to control the blood pressure and cardiac responses, but our patient also has asthma, and she is trying to get pregnant. Which of the following drugs would be the best alternative to the beta blocker in terms of likely efficacy on pressure and heart rate, and in terms of relative safety?
    a. Diltiazem
    b. Enalapril
    c. Furosemide
    d. Phentolamine
    e. Prazosin
A

a. Diltiazem

129
Q
A patient presents with what was initially thought to be Stage 2 hypertension. The actual underlying cause—a pheochromocytoma—is not looked for nor detected in the initial work-up. An oral antihypertensive drug is prescribed. We soon find that the patient's blood pressure has risen to levels above pretreatment levels—so much so that we are worried about imminently dangerous effects from the drug-induced worsening of hypertension.
Concomitant with the drug-induced rise of blood pressure the patient develops signs and symptoms of heart failure. Which of the following drugs was most likely administered?
a. Captopril
b. Hydrochlorothiazide
c. Labetalol
d. Losartan
e. Propranolol
f. Verapamil
A

e. Propranolol

130
Q

A patient with hypertension and heart failure has been treated for 2 years with carvedilol and lisinopril. He has just had hip replacement surgery, and because he is not ambulating, he is started on unfractionated heparin, postoperatively, for prophylaxis of deep venous thrombosis. Oral antacids and ranitidine (H2 antagonist) have been added for prophylaxis of acute stress ulcers. Five days postop he experiences sudden onset dyspnea and electrocardiographic and other indications of an acute MI. The patient’s platelet counts are dangerously low. Which of the following is the most likely underlying problem?

a. Accidental substitution of low-molecular-weight heparins (LMWH) for unfractionated heparin
b. Accidental/inadvertent aspirin administration
c. Hemolytic anemia from a carvedilol-ACE inhibitor interaction
d. Heparin-induced thrombocytopenia
e. Reduced heparin effects by increased metabolic clearance (caused by ranitidine)

A

d. Heparin-induced thrombocytopenia

131
Q

For many hypertensive patients we can prescribe either lisinopril (or an alternative in the same class) or losartan. Which one of the following statements correctly summarizes how losartan differs from lisinopril or its related drugs?

a. Lisinopril competitively blocks catecholamine-mediated vasoconstriction, losartan does not
b. Lisinopril effectively inhibits synthesis of Angiotensin II, losartan does not
c. Losartan causes a higher incidence of bronchospasm and hyperuricemia
d. Losartan is preferred for managing hypertension during pregnancy, whereas captopril is contraindicated
e. Losartan is suitable for administration to patients with heart failure, whereas captopril and related drugs should be avoided

A

b. Lisinopril effectively inhibits synthesis of Angiotensin II, losartan does not

132
Q

A 46-year-old man has Stage II essential hypertension, primary hypercholesterolemia, and modestly elevated fasting glucose levels (130 mg/dL) measured on several occasions. His cholesterol levels (total, HDL, LDL) have not been acceptably modified by dietary changes and daily use of a “statin.” The physician adds ezetimibe to the regimen. Which of the following statements is correct about ezetimibe’s actions, or what might be expected in response to its use?
a. Exerts profound cardiac negative inotropic effects that pose a risk of heart block
b. Frequently causes orthostatic hypotension that in turn triggers reflex cardiac stimulation
c. More likely than other drugs to increase the risk of severe statin-induced myopathy
d. Reduces intestinal cholesterol uptake, has no direct hepatic effect to inhibit
cholesterol synthesis
e. Significantly increases risk of atherosclerotic plaque rupture

A

d. Reduces intestinal cholesterol uptake, has no direct hepatic effect to inhibit
cholesterol synthesis

133
Q

***A patient presents in the emergency department with acute hypotension that requires treatment. Hypovolemia is ruled out as a cause or contributor,
and information gathered from the patient and family indicates the cause is overdose of an antihypertensive drug.
One approach to treatment is to administer a pharmacologic (ordinarily effective) dose of phenylephrine, an alpha-adrenergic agonist. You do just that, and blood pressure fails to rise at all—and a second dose doesn’t work either.
On which antihypertensive drug did the patient most likely overdose?
a. Captopril or another ACE inhibitor
b. Hydralazine
c. Prazosin
d. Thiazide diuretic (e.g., hydrochlorothiazide)
e. Verapamil

A

c. Prazosin

134
Q

***An elderly male patient who has just been referred to your practice has been taking a drug for symptomatic relief of benign prostatic hypertrophy.
In addition to its effects on smooth muscles of the prostate and urethra, this drug can lower blood pressure in such a way that it reflexly triggers tachycardia, positive inotropy, and increased AV nodal conduction. The drug neither dilates nor constricts the bronchi. It causes the pupils of the eyes to constrict and interferes with mydriasis in dim light. Initial oral dosages of this drug have been associated with a high incidence of syncope. Which prototype is most similar to this unnamed drug in terms of the pharmacologic profile?
a. Captopril
b. Hydrochlorothiazide (prototype thiazide diuretic)
c. Labetalol
d. Nifedipine
e. Prazosin
f. Propranolol
g. Verapamil

A

e. Prazosin

135
Q

You are contemplating starting ACE inhibitor therapy for a patient with essential hypertension. Which of the following patient-related condition(s) contraindicates use of an ACE inhibitor and so should be ruled out before you prescribe this drug?

a. Asthma
b. Heart failure
c. Hyperlipidemia, coronary artery disease
d. Hypokalemia
e. Is a woman who is pregnant or may become pregnant

A

e. Is a woman who is pregnant or may become pregnant

136
Q

A patient develops sinus bradycardia. Heart rate is dangerously low, and an effective and safe drug needs to be given right away. Which of the following would be the best choice?

a. Atropine
b. Amiodarone
c. Edrophonium
d. Lidocaine
e. Phentolamine

A

a. Atropine

to block excessive parasympathetic influences

137
Q

We administer reserpine on a daily basis, at usual effective doses, for a time sufficient for the drug to exert its maximal and expected effects.
Which of the following effects would most likely occur?
a. Accumulation of NE in synapses, leading to vasoconstriction and tachycardia
b. Depletion of intraneuronal NE stores leading to reduced blood pressure, heart rate, and contractility
c. Dry mouth, blurred vision, constipation, and urinary retention
d. Further metabolism of NE to epinephrine, development of a pheochromocytomalike tumor
e. Subsensitivity of pre- and post-synaptic alpha-adrenergic receptors

A

b. Depletion of intraneuronal NE stores leading to reduced blood pressure, heart rate, and contractility

138
Q
  • **A patient presents in the emergency department (ED) with severe angina pectoris, and acute myocardial ischemia is confirmed by electrocardiographic and other clinical indicators. Unknown to the ED team is the fact that the ischemia is due to coronary vasospasm, not to coronary occlusion with thrombi. Given this etiology, which one of the following drugs, administered in usually effective doses, may actually make the vasospasm, and the resulting ischemia, worse?
    a. Alteplase (t-PA)
    b. Aspirin
    c. Captopril
    d. Nitroglycerin
    e. Propranolol
    f. Verapamil
A

e. Propranolol

139
Q

A patient with essential hypertension has been treated with a fixed-dosecombination product that contains hydrochlorothiazide and triamterene.
Blood pressure and serum electrolyte profiles have been kept within acceptable limits for the last 18 months. Now, however, blood pressure has risen to
the point where the physician wants to add a third antihypertensive drug. The drug is started; after several weeks blood pressure falls into an acceptable
range, but the patient has become hyperkalemic. Which one of the following drugs was added and was most likely responsible for the desired blood pressure
fall and the unwanted rise of serum potassium levels.
a. Diltiazem
b. Prazosin
c. Propranolol
d. Ramipril
e. Verapamil

A

d. Ramipril

(Ramipril is an ACE inhibitor that ultimately will lower serum aldosterone levels which will counteract the sodium-retaining
effects of aldosterone, and also its potassium-wasting effects)

140
Q

A patient has supraventricular tachycardia. We inject a drug and
heart rate falls to a normal or at least more acceptable level. Although this drug caused the desired response, it did so without any direct effect in or on the heart. Which of the following drugs was most likely used?
a. Edrophonium
b. Esmolol
c. Phenylephrine
d. Propranolol
e. Verapamil

A

c. Phenylephrine

baroreceptor reflex

141
Q
A 69-year-old man presents with NYHA Stage II ("mild") heart failure.
He is placed on usual therapeutic doses of digoxin and furosemide. At a follow-up exam 3 months later we find good symptomatic relief of the heart failure. Serum electrolytes and all other lab tests are within normal limits. At this time, which electrocardiographic change would you expect to see in response to the digoxin's expected effects, compared with a baseline
(pretreatment) EKG?
a. P waves widened, amplitude increased
b. P-R intervals prolonged
c. QRS complexes widened
d. R-R intervals shortened
e. S-T segments elevated
A

b. P-R intervals prolonged

142
Q

A 23-year-old nonpregnant woman has been using a preparation of oral ergotamine to manage her frequent migraine headaches. She consumes an excessive dose of the drug while trying to abort a particularly severe and refractory attack. Which of the following adverse cardiac or cardiovascular consequences are most likely to occur as a result of the ergot overdose?

a. Myocardial and peripheral (e.g., limb) ischemia due to intense vasoconstriction
b. Renal failure secondary to rhabdomyolysis
c. Spontaneous bleeding due to direct inhibition of platelet activation/aggregation
d. Syncope secondary to acute hypotension
e. Tachycardia, tachyarrhythmias from alpha1 adrenergic receptor activation

A

a. Myocardial and peripheral (e.g., limb) ischemia due to intense vasoconstriction

143
Q

A patient presents with severe hypertension (BP is 220/120) and tachycardia (96 beats per minute) despite usually effective antihypertensive drug therapy. Further work-up indicates the patient has a rare cause of these and other signs and symptoms: pheochromocytoma. You realize that beta-adrenergic blockers are useful as antihypertensive drugs, and for helping to normalize heart rate in patients with sinus tachycardia. As a result of the diagnosis, and your knowledge, you administer a usually effective dose of
a nonselective (b1/b2) beta blocker. Which of the following is the most likely outcome of doing this?
a. Blood pressure falls promptly, followed by reflex tachycardia
b. Epinephrine release from the tumor is suppressed, hemodynamics normalize
c. Heart rate and cardiac function rise quickly because the beta blocker has triggered additional epinephrine release from the tumor
d. Left ventricular afterload is decreased; cardiac output rises via increases of both left ventricular stroke volume and heart rate
e. Total peripheral resistance rises, cardiac output falls, the patient goes into heart failure

A

e. Total peripheral resistance rises, cardiac output falls, the patient goes into heart failure

144
Q

A 65-year-old normotensive woman is transferred to the thoracic surgery ICU after cardiac surgery. She has diffuse rales bilaterally, a pulse of 110/min, an elevated venous pressure, and a blood pressure of 160/98 mm Hg.
The surgery resident wants to inject an otherwise correct dose of an IV drug to control heart rate and blood pressure, but grabs a syringe that contains the wrong drug. The patient’s heart rate increases to 150/min and her blood pressure rises to 180/106 mm Hg.
Which one of the following drugs was most likely given?
a. Dobutamine
b. Esmolol
c. Neostigmine
d. Propranolol
e. Verapamil

A

a. Dobutamine

145
Q

You are in a debate with a colleague over which alpha blocker to use, adjunctively, to control blood pressure in a pheochromocytoma patient.
Your colleague correctly states that phenoxybenzamine is the “preferred” drug. You state that prazosin would be a better choice. Which of the following statements about prazosin is correct in comparison with phenoxybenzamine, and might actually support your proposal that it would be the better choice?
a. Causes not only peripheral a-blockade but also suppresses adrenal epinephrine release
b. Has a longer duration of action, which enables less frequent dosing
c. Has good intrinsic beta-blocking activity, phenoxybenzamine does not
d. Overdoses, and the hypotension it may cause, are easier to manage pharmacologically
e. Will not cause orthostatic hypotension, which is a common consequence of phenoxybenzamine

A

d. Overdoses, and the hypotension it may cause, are easier to manage pharmacologically

146
Q

A man has an aneurysm in the aortic root, a consequence of Marfan’s syndrome. He experiences a hypertensive crisis that requires prompt blood pressure control. Nitroprusside will be infused for its immediate antihypertensive effects. Which of the following drugs must we administer along with the nitroprusside to minimize the risk of aneurysm rupture as blood pressure falls?

a. Atropine
b. Diazoxide
c. Furosemide
d. Phentolamine
e. Propranolol

A

e. Propranolol

prevent baroreceptor reflex

147
Q

For 9 months, a 34-year-old woman has been experiencing left-sided pulsatile headaches. Before each headache attack, she feels disoriented and sees bilateral flashes of light. The pain is always unilateral and is often associated with nausea, vomiting, and photophobia. Which of the following is the most appropriate medication for headache prophylaxis for this patient?

a) Ergonovine
b) Dihydroergotamine
c) Ondansetron
d) Propranolol
e) Sumatriptan

A

d) Propranolol

148
Q
Although not strictly "alternative medicine" the incubation of a west indian centipede in alchohol for several weeks is alleged to result in the formation of a compound that has effectiveness in erectile dysfunction. If this compound is similar to sildenafil and inhibits phosphodiesterases, it would be contraindicated in a patient who is being treated with 
A. Propanolol 
B. Hydrochlorthiazide 
C. Isosorbide dinitrate 
D. Lovastatin 
E. Amiodarone
A

C. Isosorbide dinitrate

Sildenafil is contraindicated by nitrates

149
Q

A 71 year old man has a lengthy history of coronary ischemia but has had minimal chest pain since his coronary bypass surgery 4 years ago; however, he does require medications to control his blood pressure nd to treat his benign prostatic hyperplasia. He also occasionally takes vardenafil to improve erectile dysfunction. Which of the following medications is most appropriate for this patient to take if he continues to use vardenafil?

A. Terazosin
B. Doxazosin
C. Nitroglycerin 
D. Erythromycin 
E. Tamsulosin
A

E. Tamsulosin

Only with “denafil”
Alpha blockers for BPH - “osins”

150
Q

A 60 year old patient presents to the office with complaints of chest pain. During the conversation, the patient indicates that he is under a lot of stress at his job. His blood pressure is 175/102 mmHg, pulse is 84bpm. ECG showed slightly negative T-waves in II, III, and aVF. You decide to intiate the treatment with metoprolol-XL, 50mg a day instructing the patient not to stop the medication abrupty without doctor’s supervision. Out of the following, what described the best reason behind your warning?

A. Decrease of after load
B. AV nodal depression
C. Precipitation of ischemic events 
D. Increase of myocardial relaxation 
E. Inefficient therapeutic response 
F. Undesirable negative inotropic action
A

C. Precipitation of ischemic events

151
Q

A patient comes to your office complaining of intermittent chest pain. After a number of tests you determine that he has angina and you prescibe nitroglycerin for him. Nitroglycerin is an effective vasodilator because it slowly breaks down yielding nitric oxide. The nitric oxide produced acts to stimulate which of the following enzymes?

A. adenylate cyclase 
B. guanylate cyclase 
C. cyclic GMP-dependent protein kinse 
D. protein kinase C 
E. cyclic AMP-dependent protein kinase
A

B. guanylate cyclase

152
Q

A 75 year old patient sufering from congestive heart failure accidentally ingests a toxic dose of digoxin. Clinical consequences due to the toxic effects of cardiac glycosides are least likely to include

A. Visual disturbances 
B. Hypokalemia 
C. Bigeminy 
D. Premature ventricular beats 
E. Nausea and vomiting
A

B. Hypokalemia

Least likely side effect of digoxin

153
Q

A 64 year old male with arteriosclerotic heart disease (AHD) and CHF who has been treated with digoxin complains of nausea, vomiting, and diarrhea. His EKG reveals a bigeminal rhythm. The symptoms and EKG findings occurred shortly after another therapeutic agent was added to his regimen. A drug-drug interaction is suspected. Which agent was involved?

A. Furosemide 
B. Lovastatin 
C. Phenobarbital 
D. Nitroglycerin 
E. Captopril
A

A. Furosemide

Loop diuretic

154
Q

We have a patient who is diagnosed with variant (vasospastic) angina. Which of the following drugs would be most appropriate, and generally regarded as most effective, for long-term therapy aimed at reducing the incidence or severity of the coronary vasospasm?

A. IV Nitroglycerin 
B. Atorvastatin 
C. Diltiazem 
D. Propanolol
E. Aspirin
A

C. Diltiazem

155
Q

A patient with heart failure, Stage 2 essential hypertension, and hyperlipidemia (elevated LDL cholesterol and abnormally low HDL-C) is taking furosemide, captopril, atenolol, and simvastatin (an HMG CoA reductase inhibitor).
During a scheduled physical exam, about a month after starting all the above drugs, the patient reports a severe, hacking, and relentless cough. Other vital signs, and the overall physical assessment, are consistent with good control of both the heart failure and blood pressure and indicate no other underlying disease or abnormalities. Results of blood tests are not yet available.
Which of the following is the most likely cause of the cough?
a. An expected side effect of the captopril
b. An allergic reaction to the statin
c. Dyspnea due to captopril’s known and powerful bronchoconstrictor action
d. Excessive doses of the bumetanide, which led to hypovolemia
e. Hyperkalemia caused by an interaction between bumetanide and captopril
f. Pulmonary edema from the bumetanide

A

a. An expected side effect of the captopril(ace inhibitor)

156
Q
  • *A patient, who has excessively slow AV nodal conduction rates that unfortunately haven’t been recognized, is started on a drug. As soon as blood levels climb towards the usual therapeutic range the patient goes into complete heart block. Which of the following drugs most likely provoked this further prolongation of the P-R interval, ultimately leading to the complete heart block?
    a. Captopril
    b. Losartan
    c. Nifedipine
    d. Nitroglycerin
    e. Prazosin
    f. Diltiazem
A

F. Diltiazem

or Verapamil

157
Q

In treating an elderly male patient with long standing hypertension and mild congestive heart failure, which of the following agents would be most likely to slow the progression of heart failure?

A. theophylline 
B. Lisinopril 
C. Thiazide Diuretics 
D. Dobutamine
E. Digoxin
A

B. Lisinopril

Ace Inhibitors

158
Q

For many hypertensive patients we can prescribe either lisinopril (or an alternative in the same class) or losartan. Which one of the following statements correctly summarizes how losartan differs from lisinopril or its related drugs?

a. Lisinopril competitively blocks catecholamine-mediated vasoconstriction, losartan does not
b. Lisinopril effectively inhibits synthesis of Angiotensin II, losartan does not
c. Losartan causes a higher incidence of bronchospasm and hyperuricemia
d. Losartan is preferred for managing hypertension during pregnancy, whereas captopril is contraindicated
e. Losartan is suitable for administration to patients with heart failure, whereas captopril and related drugs should be avoided

A

b. Lisinopril effectively inhibits synthesis of Angiotensin II, losartan does not

159
Q

A patient has been taking one of the drugs listed below for about 4 months and is experiencing the desired therapeutic effects from it. The MD now prescribes indomethacin to treat a particularly severe flare-up of rheumatoid arthritis. Within a matter of days the therapeutic effects of the first drug wane dramatically, its actions antagonized by the indomethacin. Which of the following was the most likely drug affected by the indomethacin?

a. Allopurinol, given for prophylaxis of hyperuricemia
b. Captopril, given for essential hypertension
c. Fexofenadine, given for managing seasonal allergy responses
d. Sumatriptan, given for abortive therapy of migraine headaches
e. Warfarin given for prophylaxis of venous thrombosis

A

b. Captopril, given for essential hypertension

NSAIDs decrease the effectiveness of loop and ace inhibitors

160
Q

Two years later, Mr. Green returns complaining that his nitroglycerin works well when he takes it for an acute attack but that he is having frequent attacks now and would like something to prevent them. Useful drugs for the prophylaxis of angina of effort include which one of the following?

A. Sublingual nitroglyceride 
B. Esmolol 
C. Sublingual Insosorbide Dinitrate 
D. Amylnitrite 
E. Diltiazem
A

E. Diltiazem

(Ca+ Channel blockers)

esmolol - shot
sublingual only acute

161
Q

A 61-year-old female has intermittent bouts of chest pain on exertion of two months’ duration, associated with numbness and tingling in the fourth and fifth fingers of her left hand. An EKG is normal. She is placed on propranolol, which relieves her symptoms. What cardiovascular effect did the drug have?
A. It decreased production of catecholamines
B. It dilated the coronary vasculature
C. It decreased the requirement for myocardial oxygen
D. It increased peripheral vascular resistance
E. It increased sensitivity to catecholamines

A

C. It decreased the requirement for myocardial oxygen

162
Q

Following an acute myocardial infarction, a patient develops signs of pulmonary edema requiring drug management. What effect would amrinone, digoxin, and a high dose dopamine have in common if each was administered individually to the patient?

A. decreased AV conduction 
B. decreasd venous return 
C. decreased PVR 
D. increased ventricular contractility 
E. Decreased cAMP
A

D. increased ventricular contractility

all positive inotropic on heart

163
Q

a 59 year old man with a history of myocardial infarction presents to his physician complaining of shortness of breath, on examination, his heart rte is 110/min and respiratory rate is 22/min. He has rales in both lung fields, a normal sinus rhythm with an S3 gallop, and 2+ pitting edema. A chest xray film reveals cardiomegaly, and his rejection fraction on echocardiogram is calculated at 37%. Which of the following medications would alleviate this patient’s symptoms by significantly reducing both the preload and after load on the heart without affecting its inotropic state?

A. Propanolol 
B. Digoxin 
C. Enalapril 
D. Furosemide 
E. Dilitazem
A

C. Enalapril

S3 gallop CHF - Ace inhibitors

164
Q

A man has an aneurysm in the aortic root, a consequence of Marfan’s syndrome. He experiences a hypertensive crisis that requires prompt blood pressure control. Nitroprusside will be infused for its immediate antihypertensive effects. Which of the following drugs must we administer along with the nitroprusside to minimize the risk of aneurysm rupture as blood pressure falls?

a. Atropine
b. Diazoxide
c. Furosemide
d. Phentolamine
e. Propranolol

A

e. Propranolol

for baroreceptor reflex

165
Q

A 61 year old man with erectile dysfunction asks his physician to prescibe Viagra (sildenafil ciltrate). Sildenafil produces its physiological effects by blocking the enzyme that hydrolyzes the second messenger by which nitric oxide produces its physiological effects. Which of the following is the second messenger?

a. G protein
b. Cyclic GMP
c. Diacylglycerol
d. Guanylate cyclase
e. cGMP phosphodiesterase

A

b. Cyclic GMP

166
Q

Calcium channel blockers are commonly used cardiovascular drugs that are effective antihypertensives, antianginals, and antiarrhythmics. Diltiazem is more effective than nifedipine for cardiac indications because diltiazem slows the recovery of the slow calcium channel while also reducing the influx of calcium into the myocyte. Which of the following is an effect of diltiazem?

A. Increase in cardiac inotropy 
B. Decrease in cardiac chronotropy 
C. Increase in conduction velocity 
D. Increase in vascular smooth muscle tone 
E. All of the above
A

B. Decrease in cardiac chronotropy

167
Q

A patient has received excessive doses of nitroprusside, and toxic manifestations are developing. Which of following drugs should we administer to help nitroprusside’s metabolism proceed to the formation of a less toxic metabolite?

A. Vitamin C 
B. Thrombin 
C. Sodium thiosulfate 
D. Epinephrine 
E. Vitamin E 
F. Vitamin K
A

C. Sodium thiosulfate